urinary Flashcards

1
Q
  1. Which of the following solutions, when infused intravenously, would result in an increase in extracellular
    fluid volume, a decrease in intracellular fluid volume,
    and an increase in total body water after osmotic
    equilibrium?
    A) 1 Liter of 0.9% sodium chloride (NaCl) solution
    B) 1 Liter of 0.45% NaCl solution
    C) 1 Liter of 3% NaCl solution
    D) 1 Liter of 5% dextrose solution
    E) 1 Liter of pure water
A

C) 1 Liter of 3% NaCl solution

How well did you know this?
1
Not at all
2
3
4
5
Perfectly
2
Q
  1. Partial obstruction of a major vein draining a tissue would tend to __________ lymph flow rate,
    __________ interstitial fluid hydrostatic pressure, and
    __________ interstitial fluid protein concentration in
    the tissue drained by that vein.
    A) Increase, increase, increase
    B) Increase, increase, decrease
    C) Increase, decrease, decrease
    D) Decrease, decrease, decrease
    E) Decrease, increase, increase
    F) Decrease, increase, decrease
A

B) Increase, increase, decrease

How well did you know this?
1
Not at all
2
3
4
5
Perfectly
3
Q
  1. A 36-year-old woman reports headaches and frequent
    urination. Laboratory values reveal the following information.
    Urine specific gravity = 1.003
    Urine protein = negative
    Plasma sodium (Na+) = 165 mmol/L
    Plasma potassium (K+) = 4.4 mmol/L
    Plasma creatinine = 1.4 mg/dl
    Blood pressure = 88/40 mm Hg
    Heart rate = 115 beats/min
    What is the most likely cause of her elevated plasma
    Na+ concentration?
    A) Primary aldosteronism
    B) Diabetes mellitus
    C) Diabetes insipidus
    D) Simple dehydration due to insufficient water
    intake and heavy exercise
    E) Bartter’s syndrome
    F) Liddle’s syndrome
A

C) Diabetes insipidus

How well did you know this?
1
Not at all
2
3
4
5
Perfectly
4
Q
  1. After receiving a kidney transplant, a patient has
    severe hypertension (170/110 mm Hg). A renal
    arteriogram indicates severe renal artery stenosis in his
    single remaining kidney, with a reduction in glomerular
    filtration rate (GFR) to 25% of normal. Which of the following changes, compared with normal, would be expected in this patient, assuming steady-state conditions?
    A) A large increase in plasma sodium concentration
    B) A reduction in urinary sodium excretion to 25% of
    normal
    C) A reduction in urinary creatinine excretion to 25%
    of normal
    D) An increase in serum creatinine to about four
    times normal
    E) Normal renal blood flow in the stenotic kidney
    due to autoregulation
A

D) An increase in serum creatinine to about four

times normal

How well did you know this?
1
Not at all
2
3
4
5
Perfectly
5
Q
  1. Which of the following tends to decrease potassium
    secretion by the cortical collecting tubule?
    A) Increased plasma potassium concentration
    B) A diuretic that decreases proximal tubule sodium
    reabsorption
    C) A diuretic that inhibits the action of aldosterone
    (e.g., spironolactone)
    D) Acute alkalosis
    E) High sodium intake
A

C) A diuretic that inhibits the action of aldosterone

How well did you know this?
1
Not at all
2
3
4
5
Perfectly
6
Q
  1. Because the usual rate of phosphate filtration exceeds
    the transport maximum for phosphate reabsorption,
    which statement is true?
    A) All the phosphate that is filtered is reabsorbed
    B) More phosphate is reabsorbed than is filtered
    C) Phosphate in the tubules can contribute significantly to titratable acid in the urine
    D) The “threshold” for phosphate is usually not
    exceeded
    E) Parathyroid hormone must be secreted for phosphate reabsorption to occur
A

C) Phosphate in the tubules can contribute significantly to titratable acid in the urine

How well did you know this?
1
Not at all
2
3
4
5
Perfectly
7
Q
  1. If a patient has a creatinine clearance of 90 ml/min,
    a urine flow rate of 1 ml/min, a plasma K+ concentration of 4 mEq/L, and a urine K+ concentration
    of 60 mEq/L, what is the approximate rate of K+
    excretion?
    A) 0.06 mEq/min
    B) 0.30 mEq/min
    C) 0.36 mEq/min
    D) 3.6 mEq/min
    E) 60 mEq/min
A

A) 0.06 mEq/min

How well did you know this?
1
Not at all
2
3
4
5
Perfectly
8
Q
  1. Given the following measurements, calculate the filtration fraction:
    Glomerular capillary hydrostatic pressure (PG) = 70 mm
    Hg
    Bowman’s space hydrostatic pressure (PB) = 20 mm Hg
    Colloid osmotic pressure in the glomerular capillaries
    (πG) = 35 mm Hg
    Glomerular capillary filtration coefficient (Kf
    ) = 10 ml/
    min/mm Hg
    Renal plasma flow = 428 ml/min
    A) 0.16
    B) 0.20
    C) 0.25
    D) 0.30
    E) 0.35
    F) 0.40
A

E) 0.35

How well did you know this?
1
Not at all
2
3
4
5
Perfectly
9
Q
  1. In normal kidneys, which of the following is true of the
    osmolarity of renal tubular fluid that flows through the
    early distal tubule in the region of the macula densa?
    A) Usually isotonic compared with plasma
    B) Usually hypotonic compared with plasma
    C) Usually hypertonic compared with plasma
    D) Hypertonic, compared with plasma, in antidiuresis
A

B) Usually hypotonic compared with plasma

How well did you know this?
1
Not at all
2
3
4
5
Perfectly
10
Q
  1. When the dietary intake of K+ increases, body K+
    balance is maintained by an increase in K+ excretion
    primarily by which of the following?
    A) Decreased glomerular filtration of K+
    B) Decreased reabsorption of K+ by the proximal tubule
    C) Decreased reabsorption of K+ by the thick ascending
    limb of the loop of Henle
    D) Increased K+ secretion by the late distal and
    collecting tubules
    E) Shift of K+ into the intracellular compartment
A

D) Increased K+ secretion by the late distal and

collecting tubules

How well did you know this?
1
Not at all
2
3
4
5
Perfectly
11
Q
  1. Which of the following would cause the greatest decrease
    in GFR in a person with otherwise normal kidneys?
    A) Decrease in renal arterial pressure from 100 to 80
    mm Hg in a normal kidney
    B) 50% increase in glomerular capillary filtration
    coefficient
    C) 50% increase in proximal tubular sodium reabsorption
    D) 50% decrease in afferent arteriolar resistance
    E) 50% decrease in efferent arteriolar resistance
    F) 5 mm Hg decrease in Bowman’s capsule pressur
A

E) 50% decrease in efferent arteriolar resistance

How well did you know this?
1
Not at all
2
3
4
5
Perfectly
12
Q
  1. The clinical laboratory returned the following values
    for arterial blood taken from a patient: plasma pH =
    7.28, plasma HCO3
    − = 32 mEq/L, and plasma partial
    pressure of carbon dioxide (Pco2) = 70 mm Hg. What
    is this patient’s acid-base disorder?
    A) Acute respiratory acidosis without renal compensation
    B) Respiratory acidosis with partial renal compensation
    C) Acute metabolic acidosis without respiratory
    compensation
    D) Metabolic acidosis with partial respiratory compensation
A

B) Respiratory acidosis with partial renal compensation

How well did you know this?
1
Not at all
2
3
4
5
Perfectly
13
Q
22. The following laboratory values were obtained in a 
58-year-old man:
Urine volume = 4320 milliliters of urine collected 
during the preceding 24 hours
Plasma creatinine = 3 mg/100 ml
Urine creatinine = 50 mg/100 ml
Plasma potassium = 4.0 mmol/L
Urine potassium = 30 mmol/L
What is his approximate GFR, assuming that he 
collected all of his urine in the 24-hour period?
A) 20 ml/min
B) 30 ml/min
C) 40 ml/min
D) 50 ml/min
E) 60 ml/min
F) 80 ml/min
G) 100 ml/min
A

D) 50 ml/min

How well did you know this?
1
Not at all
2
3
4
5
Perfectly
14
Q
  1. What would cause the greatest degree of hyperkalemia?
    A) Increase in potassium intake from 60 to 180
    mmol/day in a person with normal kidneys and a
    normal aldosterone system
    B) Chronic treatment with a diuretic that inhibits the
    action of aldosterone
    C) Decrease in sodium intake from 200 to 100 mmol/
    day
    D) Chronic treatment with a diuretic that inhibits
    loop of Henle Na+-2Cl−-K+ co-transport
    E) Chronic treatment with a diuretic that inhibits sodium reabsorption in the collecting ducts
A

B) Chronic treatment with a diuretic that inhibits the

action of aldosterone

How well did you know this?
1
Not at all
2
3
4
5
Perfectly
15
Q
  1. Which change tends to increase GFR?
    A) Increased afferent arteriolar resistance
    B) Decreased efferent arteriolar resistance
    C) Increased glomerular capillary filtration coefficient
    D) Increased Bowman’s capsule hydrostatic pressure
    E) Decreased glomerular capillary hydrostatic pressure
A

C) Increased glomerular capillary filtration coefficient

How well did you know this?
1
Not at all
2
3
4
5
Perfectly
16
Q
  1. What is the net renal tubular reabsorption rate of
    potassium in the patient described in Question 5?
    A) 0.020 mmol/min
    B) 0.040 mmol/min
    C) 0.090 mmol/min
    D) 0.110 mmol/min
    E) 0.200 mmol/min
    F) Potassium is not reabsorbed in this example
A

D) 0.110 mmol/min

How well did you know this?
1
Not at all
2
3
4
5
Perfectly
17
Q
  1. The maximum clearance rate possible for a substance
    that is totally cleared from the plasma is equal to
    which of the following?
    A) GFR
    B) Filtered load of that substance
    C) Urinary excretion rate of that substance
    D) Renal plasma flow
    E) Filtration fraction
A

D) Renal plasma flow

How well did you know this?
1
Not at all
2
3
4
5
Perfectly
18
Q
  1. A patient has the following laboratory values: arterial
    pH = 7.13, plasma HCO3
    − = 15 mEq/L, plasma chloride concentration = 118 mEq/L, arterial Pco2 = 28
    mm Hg, and plasma Na+ concentration = 141 mEq/L.
    What is the most likely cause of his acidosis?
    A) Salicylic acid poisoning
    B) Diabetes mellitus
    C) Diarrhea
    D) Emphysema
A

C) Diarrhea

How well did you know this?
1
Not at all
2
3
4
5
Perfectly
19
Q
34. The GFR of a 26-year-old man with glomerulonephritis decreases by 50% and remains at that level. For 
which substance would you expect to find the greatest 
increase in plasma concentration?
A) Creatinine
B) K+
C) Glucose
D) Na+
E) Phosphate
F) H+
A

A) Creatinine

How well did you know this?
1
Not at all
2
3
4
5
Perfectly
20
Q

Questions 35 and 36
Assume the following initial conditions: intracellular fluid
volume = 40% of body weight before fluid administration,
extracellular fluid volume = 20% of body weight before fluid
administration, molecular weight of NaCl = 58.5 g/mol,
and no excretion of water or electrolytes.
35. A male patient appears to be dehydrated, and
after obtaining a plasma sample, you find that he
has hyponatremia, with a plasma sodium concentration of 130 mmol/L and a plasma osmolarity
of 260 mOsm/L. You decide to administer 2 L of
3% sodium chloride (NaCl). His body weight was
60 kilograms before the fluid is administered. What
is his approximate plasma osmolarity after administration of the NaCl solution and after osmotic equilibrium? Assume the initial conditions previously
described.
A) 273 mOsm/L
B) 286 mOsm/L
C) 300 mOsm/L
D) 310 mOsm/L
E) 326 mOsm/L
36. What is the approximate extracellular fluid volume in
this patient after administration of the NaCl solution
and after osmotic equilibrium?
A) 15.1 Liters
B) 17.2 Liters
C) 19.1 Liters
D) 19.8 Liters
E) 21.2 Liters

A
  1. C) 300 mOsm/L

36. B) 17.2 Liters

How well did you know this?
1
Not at all
2
3
4
5
Perfectly
21
Q
  1. Which changes would you expect to find after administering a vasodilator drug that caused a 50% decrease
    in afferent arteriolar resistance and no change in arterial pressure?
    A) Decreased renal blood flow, decreased GFR, and decreased peritubular capillary hydrostatic pressure
    B) Decreased renal blood flow, decreased GFR, and
    increased peritubular capillary hydrostatic pressure
    C) Increased renal blood flow, increased GFR, and increased peritubular capillary hydrostatic pressure
    D) Increased renal blood flow, increased GFR, and no
    change in peritubular capillary hydrostatic pressure
    E) Increased renal blood flow, increased GFR, and decreased peritubular capillary hydrostatic pressure
A

C) Increased renal blood flow, increased GFR, and increased peritubular capillary hydrostatic pressure

How well did you know this?
1
Not at all
2
3
4
5
Perfectly
22
Q
  1. If the average hydrostatic pressure in the glomerular capillaries is 50 mm Hg, the hydrostatic pressure in the Bowman’s space is 12 mm Hg, the average colloid osmotic
    pressure in the glomerular capillaries is 30 mm Hg, and
    there is no protein in the glomerular ultrafiltrate, what is
    the net pressure driving glomerular filtration?
    A) 8 mm Hg
    B) 32 mm Hg
    C) 48 mm Hg
    D) 60 mm Hg
    E) 92 mm Hg
A

A) 8 mm Hg

How well did you know this?
1
Not at all
2
3
4
5
Perfectly
23
Q
  1. If distal tubule fluid creatinine concentration is
    5 mg/100 ml and plasma creatinine concentration is
    1.0 mg/100 ml, what is the approximate percentage
    of the water filtered by the glomerular capillaries that
    remains in the distal tubule?
    A) 5%
    B) 10%
    C) 20%
    D) 50%
    E) 80%
    F) 95%
A

C) 20%

How well did you know this?
1
Not at all
2
3
4
5
Perfectly
24
Q
  1. Which change tends to increase peritubular capillary
    fluid reabsorption?
    A) Increased blood pressure
    B) Decreased filtration fraction
    C) Increased efferent arteriolar resistance
    D) Decreased angiotensin II
    E) Increased renal blood flow
A

C) Increased efferent arteriolar resistance

How well did you know this?
1
Not at all
2
3
4
5
Perfectly
25
Q
  1. A 32-year-old man reports frequent urination. He
    is overweight (280 pounds [127 kilograms], 5 feet 10
    inches [178 cm] tall). After measuring the 24-hour creatinine clearance, you estimate his GFR to be 150 ml/
    min. His plasma glucose level is 300 mg/dl. Assuming
    that his renal transport maximum for glucose is normal, as shown in the figure above, what would be this
    patient’s approximate rate of urinary glucose excretion?
    A) 0 mg/min
    B) 100 mg/min
    C) 150 mg/min
    D) 225 mg/min
    E) 300 mg/min
    F) Information provided is inadequate to estimate
    the glucose excretion rate
A

C) 150 mg/min

How well did you know this?
1
Not at all
2
3
4
5
Perfectly
26
Q
  1. An adrenal tumor that causes excess aldosterone secretion would tend to __________ plasma K+ concentration,
    __________ plasma pH, __________ renin secretion, and
    __________ blood pressure.
    A) Decrease, decrease, decrease, decrease
    B) Decrease, increase, decrease, increase
    C) Decrease, decrease, decrease, increase
    D) Decrease, increase, increase, increase
    E) Increase, increase, decrease, increase
    F) Increase, decrease, decrease, increase
A

B) Decrease, increase, decrease, increase

How well did you know this?
1
Not at all
2
3
4
5
Perfectly
27
Q
  1. Which of the following tends to increase potassium
    secretion by the cortical collecting tubule?
    A) A diuretic that inhibits the action of aldosterone
    (e.g., spironolactone)
    B) A diuretic that decreases loop of Henle sodium
    reabsorption (e.g., furosemide)
    C) Decreased plasma potassium concentration
    D) Acute metabolic acidosis
    E) Low sodium intake
A

B) A diuretic that decreases loop of Henle sodium

reabsorption (e.g., furosemide)

How well did you know this?
1
Not at all
2
3
4
5
Perfectly
28
Q
  1. A 62-year-old woman has previously had a unilateral
    nephrectomy after diagnosis of renal carcinoma. Her
    GFR (estimated from creatinine clearance) is 50 ml/min,
    her urine flow rate is 2.0 ml/min, and her plasma glucose concentration is 200 mg/100 ml. If she has a kidney
    transport maximum for glucose of 150 mg/min, what
    would be her approximate rate of glucose excretion?
    A) 0 mg/min
    B) 50 mg/min
    C) 100 mg/min
    D) 150 mg/min
    E) 200 mg/min
    F) 300 mg/min
    G) Glucose excretion rate cannot be estimated from
    these data
A

A) 0 mg/min

How well did you know this?
1
Not at all
2
3
4
5
Perfectly
29
Q
  1. A 48-year-old woman reports severe polyuria (producing about 0.5 liter of urine each hour) and polydipsia
    (drinking two to three glasses of water every hour). Her
    urine contains no glucose, and she is placed on overnight
    water restriction for further evaluation. The next morning, she is weak and confused, her sodium concentration
    is 160 mEq/L, and her urine osmolarity is 80 mOsm/L.
    Which of the following is the most likely diagnosis?
    A) Diabetes mellitus
    B) Diabetes insipidus
    C) Primary aldosteronism
    D) Renin-secreting tumor
    E) Syndrome of inappropriate ADH
A

B) Diabetes insipidus

How well did you know this?
1
Not at all
2
3
4
5
Perfectly
30
Q
  1. Which substance is filtered most readily by the
    glomerular capillaries?
    A) Albumin in plasma
    B) Neutral dextran with a molecular weight of 25,000
    C) Polycationic dextran with a molecular weight of
    25,000
    D) Polyanionic dextran with a molecular weight of
    25,000
    E) Red blood cells
A

C) Polycationic dextran with a molecular weight of

25,000

How well did you know this?
1
Not at all
2
3
4
5
Perfectly
31
Q
  1. A 22-year-old woman runs a 10-kilometer race on a
    hot day and becomes dehydrated. Assuming that her
    ADH levels are very high and that her kidneys are
    functioning normally, in which part of the renal tubule
    is the most water reabsorbed?
    A) Proximal tubule
    B) Loop of Henle
    C) Distal tubule
    D) Cortical collecting tubule
    E) Medullary collecting duct
A

A) Proximal tubule

How well did you know this?
1
Not at all
2
3
4
5
Perfectly
32
Q
  1. Furosemide (Lasix) is a diuretic that also produces
    natriuresis. Which of the following is an undesirable
    side effect of furosemide due to its site of action on the
    renal tubule?
    A) Edema
    B) Hyperkalemia
    C) Hypercalcemia
    D) Decreased ability to concentrate the urine
    E) Heart failure
A

D) Decreased ability to concentrate the urine

How well did you know this?
1
Not at all
2
3
4
5
Perfectly
33
Q
  1. A female patient has unexplained severe hypernatremia (plasma Na+ = 167 mmol/L) and reports frequent
    urination and large urine volumes. A urine specimen
    reveals that the Na+ concentration is 15 mmol/L (very
    low) and the osmolarity is 155 mOsm/L (very low).
    Laboratory tests reveal the following data: plasma
    renin activity = 3 ng angiotensin I/ml/h (normal = 1.0),
    plasma ADH = 30 pg/ml (normal = 3 pg/ml), and plasma
    aldosterone = 20 ng/dl (normal = 6 ng/dl). Which of
    the following is the most likely reason for her hypernatremia?
    A) Simple dehydration due to decreased water intake
    B) Nephrogenic diabetes insipidus
    C) Central diabetes insipidus
    D) Syndrome of inappropriate ADH
    E) Primary aldosteronism
    F) Renin-secreting tumor
A

B) Nephrogenic diabetes insipidus

How well did you know this?
1
Not at all
2
3
4
5
Perfectly
34
Q
  1. Which change would you expect to find in a dehydrated person deprived of water for 24 hours?
    A) Decreased plasma renin activity
    B) Decreased plasma antidiuretic hormone concentration
    C) Increased plasma atrial natriuretic peptide concentration
    D) Increased water permeability of the collecting
    duct
A

D) Increased water permeability of the collecting

duct

34
Q
  1. Which change would you expect to find in a dehydrated person deprived of water for 24 hours?
    A) Decreased plasma renin activity
    B) Decreased plasma antidiuretic hormone concentration
    C) Increased plasma atrial natriuretic peptide concentration
    D) Increased water permeability of the collecting
    duct
A

D) Increased water permeability of the collecting

duct

35
Q
  1. Juvenile (type 1) diabetes mellitus is often diagnosed
    because of polyuria (high urine flow) and polydipsia
    (frequent drinking) that occur because of which of the
    following?
    A) Increased delivery of glucose to the collecting duct
    interferes with the action of antidiuretic hormone
    B) Increased glomerular filtration of glucose increases Na+ reabsorption via the sodium-glucose
    co-transporter
    C) When the filtered load of glucose exceeds the renal threshold, a rising glucose concentration in
    the proximal tubule decreases the osmotic driving
    force for water reabsorption
    D) High plasma glucose concentration decreases thirst
    E) High plasma glucose concentration stimulates
    ADH release from the posterior pituitary
A

C) When the filtered load of glucose exceeds the renal threshold, a rising glucose concentration in
the proximal tubule decreases the osmotic driving
force for water reabsorption

36
Q
  1. Which of the following would cause the most serious
    hypokalemia?
    A) A decrease in potassium intake from 150 mEq/day
    to 60 mEq/day
    B) An increase in sodium intake from 100 to 200
    mEq/day
    C) Excessive aldosterone secretion plus high sodium
    intake
    D) Excessive aldosterone secretion plus low sodium
    intake
    E) A patient with Addison’s disease
    F) Treatment with a β-adrenergic blocker
    G) Treatment with spironolactone
A

C) Excessive aldosterone secretion plus high sodium

intake

37
Q
  1. Under conditions of normal renal function, what is
    true of the concentration of urea in tubular fluid at the
    end of the proximal tubule?
    A) It is higher than the concentration of urea in tubular fluid at the tip of the loop of Henle
    B) It is higher than the concentration of urea in the
    plasma
    C) It is higher than the concentration of urea in the
    final urine in antidiuresis
    D) It is lower than plasma urea concentration because of
    active urea reabsorption along the proximal tubule
A

B) It is higher than the concentration of urea in the

plasma

38
Q
  1. Which change, compared with normal, would be
    expected to occur, under steady-state conditions,
    in a patient whose severe renal disease has reduced the number of functional nephrons to 25% of
    normal?
    A) Increased GFR of the surviving nephrons
    B) Decreased urinary creatinine excretion rate
    C) Decreased urine flow rate in the surviving nephrons
    D) Decreased urinary excretion of sodium
    E) Increased urine-concentrating ability
A

A) Increased GFR of the surviving nephrons

39
Q
  1. Which of the following would likely lead to hyponatremia?
    A) Excessive ADH secretion
    B) Restriction of fluid intake
    C) Excess aldosterone secretion
    D) Administration of 2 liters of 3% NaCl solution
    E) Administration of 2 liters of 0.9% NaCl solution
A

A) Excessive ADH secretion

40
Q
  1. Assuming steady-state conditions and that water and
    electrolyte intake remained constant, a 75% loss of
    nephrons and a 75% decrease in GFR due to chronic
    kidney disease would cause all of the following changes except what?
    A) A large increase in plasma sodium concentration
    B) An increase in plasma creatinine to four times
    normal
    C) An increase in average volume excreted per
    remaining nephron to four times normal
    D) A significant increase in plasma phosphate
    concentration
    E) Reduced ability of the kidney to maximally
    concentrate the urine
A

A) A large increase in plasma sodium concentration

41
Q
  1. Which statement is correct?
    A) Urea reabsorption in the medullary collecting tubule is less than in the distal convoluted tubule
    during antidiuresis
    B) Urea concentration in the interstitial fluid of the
    renal cortex is greater than in the interstitial fluid
    of the renal medulla during antidiuresis
    C) The thick ascending limb of the loop of Henle
    reabsorbs more urea than the inner medullary
    collecting tubule during antidiuresis
    D) Urea reabsorption in the proximal tubule is greater than in the cortical collecting tubule
A

D) Urea reabsorption in the proximal tubule is greater than in the cortical collecting tubule

42
Q
  1. A patient’s urine is collected for 2 hours, and the total
    volume is 600 milliliters during this time. Her urine
    osmolarity is 150 mOsm/L, and her plasma osmolarity is 300 mOsm/L. What is her “free water clearance”?
    A) +5.0 ml/min
    B) +2.5 ml/min
    C) 0.0 ml/min
    D) −2.5 ml/min
    E) −5.0 ml/min
A

B) +2.5 ml/min

43
Q
70. Which substances are best suited to measure interstitial fluid volume?
A) Inulin and heavy water
B) Inulin and 22Na
C) Heavy water and 125I-albumin
D) Inulin and 125I-albumin
E) 51Cr red blood cells and 125I-albumin
A

D) Inulin and 125I-albumin

44
Q
  1. Long-term administration of furosemide (Lasix)
    would do what?
    A) Inhibit the Na+-Cl− co-transporter in the renal
    distal tubules
    B) Inhibit the Na+-Cl−-K+ co-transporter in the renal
    tubules
    C) Tend to reduce renal concentrating ability
    D) Tend to cause hyperkalemia
    E) A and C
    F) B and C
    G) B, C, and D
A

F) B and C

45
Q
  1. Which of the following would be expected to cause
    a decrease in extracellular fluid potassium concentration (hypokalemia) at least in part by stimulating potassium uptake into the cells?
    A) α-adrenergic blockade
    B) Insulin deficiency
    C) Strenuous exercise
    D) Aldosterone deficiency (Addison’s disease)
    E) Metabolic alkalosis
A

E) Metabolic alkalosis

46
Q
  1. If a person has a kidney transport maximum for glucose of 350 mg/min, a GFR of 100 ml/min, a plasma
    glucose level of 150 mg/dl, a urine flow rate of 2 ml/min,
    and no detectable glucose in the urine, what would be
    the approximate rate of glucose reabsorption, assuming
    normal kidneys?
    A) Glucose reabsorption cannot be estimated from
    these data
    B) 0 mg/min
    C) 50 mg/min
    D) 150 mg/min
    E) 350 mg/min
A

D) 150 mg/min

47
Q
75. Which diuretic inhibits Na+-2Cl−-K+ co-transport in 
the loop of Henle as its primary action?
A) Thiazide diuretic
B) Furosemide
C) Carbonic anhydrase inhibitor
D) Osmotic diuretic
E) Amiloride
F) Spironolactone
A

B) Furosemide

48
Q
  1. A selective decrease in efferent arteriolar resistance
    would __________ glomerular hydrostatic pressure,
    __________ GFR, and __________ renal blood flow.
    A) Increase, increase, increase
    B) Increase, decrease, increase
    C) Increase, decrease, decrease
    D) Decrease, increase, decrease
    E) Decrease, decrease, increase
    F) Decrease, increase, increase
A

E) Decrease, decrease, increase

49
Q
83. Which nephron segment is the primary site of magnesium reabsorption under normal conditions?
A) Proximal tubule
B) Descending limb of the loop of Henle
C) Ascending limb of the loop of Henle
D) Distal convoluted tubule
E) Collecting ducts
A

C) Ascending limb of the loop of Henle

50
Q
  1. The principal cells in the cortical collecting tubules
    A) Are the main site of action of the thiazide diuretics
    B) Have sodium-chloride-potassium co-transporters
    C) Are highly permeable to urea during antidiuresis
    D) Are an important site of action of amiloride
    E) Are the main site of action of furosemide
A

D) Are an important site of action of amiloride

51
Q
  1. A patient has a GFR of 100 ml/min, her urine flow rate
    is 2.0 ml/min, and her plasma glucose concentration is
    200 mg/100 ml. If the kidney transport maximum for
    glucose is 250 mg/min, what would be her approximate rate of glucose excretion?
    A) 0 mg/min
    B) 50 mg/min
    C) 100 mg/min
    D) 150 mg/min
    E) 200 mg/min
    F) 300 mg/min
    G) Glucose excretion rate cannot be estimated from
    these data
A

A) 0 mg/min

52
Q
  1. Which change would you expect to find in a patient
    who developed acute renal failure after ingesting
    poisonous mushrooms that caused renal tubular
    necrosis?
    A) Increased plasma bicarbonate concentration
    B) Metabolic acidosis
    C) Decreased plasma potassium concentration
    D) Decreased blood urea nitrogen concentration
    E) Decreased hydrostatic pressure in Bowman’s capsule
A

D) Decreased blood urea nitrogen concentration

53
Q
  1. The type A intercalated cells in the collecting tubules
    A) Are highly permeable to urea during antidiuresis
    B) Secrete K+
    C) Secrete H+
    D) Are the main site of action of furosemide
    E) Are the main site of action of thiazide diuretics
A

C) Secrete H+

54
Q
  1. Which of the following would be the most likely
    cause of hypernatremia associated with a small volume of highly concentrated urine (osmolarity = 1400
    mOsm/L) in a person with normal kidneys?
    A) Primary aldosteronism
    B) Diabetes mellitus
    C) Diabetes insipidus
    D) Dehydration due to insufficient water intake and
    heavy exercise
    E) Bartter’s syndrome
    F) Liddle’s syndrome
A

D) Dehydration due to insufficient water intake and

heavy exercise

55
Q
  1. The most serious hypokalemia would occur in which
    condition?
    A) Decrease in potassium intake from 150 to 60 mEq/
    day
    B) Increase in sodium intake from 100 to 200 mEq/
    day
    C) Fourfold increase in aldosterone secretion plus
    high sodium intake
    D) Fourfold increase in aldosterone secretion plus
    low sodium intake
    E) Addison’s disease
A

C) Fourfold increase in aldosterone secretion plus

high sodium intake

56
Q
  1. Which of the following has similar values for both
    intracellular and interstitial body fluids?
    A) Potassium ion concentration
    B) Colloid osmotic pressure
    C) Sodium ion concentration
    D) Chloride ion concentration
    E) Total osmolarity
A

E) Total osmolarity

57
Q
  1. Which of the following is true of the tubular fluid that
    passes through the lumen of the early distal tubule in
    the region of the macula densa?
    A) It is usually isotonic
    B) It is usually hypotonic
    C) It is usually hypertonic
    D) It is hypertonic in antidiuresis
    E) It is hypertonic when the filtration rate of its own
    nephron decreases to 50% below normal
A

B) It is usually hypotonic

58
Q
  1. In a person with normal kidneys and normal lungs who
    has chronic metabolic acidosis, you would expect to
    find all of the following, compared with normal, except:
    A) Increased renal excretion of NH4Cl
    B) Decreased urine pH
    C) Decreased urine HCO3
    − excretion
    D) Increased plasma HCO3
    − concentration
    E) Decreased plasma Pco2
A

D) Increased plasma HCO3

− concentration

59
Q
  1. In a patient with very high levels of aldosterone and
    otherwise normal kidney function, approximately
    what percentage of the filtered load of sodium would
    be reabsorbed by the distal convoluted tubule and collecting duct?
    A) >66%
    B) 40% to 60%
    C) 20% to 40%
    D) 10% to 20%
    E) <10%
A

E) <10%

60
Q
  1. If the renal clearance of substance X is 300 ml/min
    and the glomerular filtration rate is 100 ml/min, it is
    most likely that substance X is
    A) Filtered freely but not secreted or reabsorbed
    B) Bound to plasma proteins
    C) Secreted
    D) Reabsorbed
    E) Bound to tubular proteins
    F) Clearance of a substance cannot be greater than
    the GFR
A

C) Secreted

61
Q
  1. Which change tends to increase urinary calcium
    (Ca++) excretion?
    A) Extracellular fluid volume expansion
    B) Increased plasma parathyroid hormone concentration
    C) Decreased blood pressure
    D) Increased plasma phosphate concentration
    E) Metabolic alkalosis
A

A) Extracellular fluid volume expansion

62
Q
  1. Which change would you expect to find in a patient
    consuming a high-sodium diet (200 mEq/day) compared with the same patient on a normal-sodium diet
    (100 mEq/day), assuming steady-state conditions?
    A) Increased plasma aldosterone concentration
    B) Increased urinary potassium excretion
    C) Decreased plasma renin activity
    D) Decreased plasma atrial natriuretic peptide
    E) An increase in plasma sodium concentration of at
    least 5 mmol/L
A

C) Decreased plasma renin activity

63
Q
  1. What would tend to decrease GFR by more than 10%
    in a normal kidney?
    A) Decrease in renal arterial pressure from 100 to
    85 mm Hg
    B) 50% decrease in afferent arteriolar resistance
    C) 50% decrease in efferent arteriolar resistance
    D) 50% increase in the glomerular capillary filtration
    coefficient
    E) Decrease in plasma colloid osmotic pressure from
    28 to 20 mm Hg
A

C) 50% decrease in efferent arteriolar resistance

64
Q
  1. Acute metabolic acidosis tends to _____ intracellular
    K+ concentration and _____ K+ secretion by the cortical collecting tubules.
    A) Increase, increase
    B) Increase, decrease
    C) Decrease, increase
    D) Decrease, decrease
    E) Cause no change in, increase
    F) Cause no change in, cause no change in
A

D) Decrease, decrease

65
Q
  1. A 55-year-old overweight male patient reports frequent urination, and his blood pressure is 165/98
    mm Hg. Based on 24-hour creatinine clearance,
    you estimate his GFR to be 150 ml/min. His plasma
    glucose is 400 mg/100 ml. Assuming that his renal
    transport maximum for glucose is normal, as shown
    in the above figure, what would be the approximate
    rate of urinary glucose excretion for this patient?
    A) 0 mg/min
    B) 100 mg/min
    C) 150 mg/min
    D) 225 mg/min
    E) 300 mg/min
    F) The information provided is inadequate to estimate the glucose excretion rate
A

E) 300 mg/min

66
Q
  1. Which statement is true?
    A) ADH increases water reabsorption from the ascending loop of Henle
    B) Water reabsorption from the descending loop of
    Henle is normally less than that from the ascending loop of Henle
    C) Sodium reabsorption from the ascending loop of
    Henle is normally less than that from the descending loop of Henle
    D) Osmolarity of fluid in the early distal tubule would
    be less than 300 mOsm/L in a dehydrated person
    with normal kidneys and increased ADH levels
    E) ADH decreases the urea permeability in the medullary collecting tubules
A

D) Osmolarity of fluid in the early distal tubule would
be less than 300 mOsm/L in a dehydrated person
with normal kidneys and increased ADH levels

67
Q
  1. In a person on a high-potassium (200 mmol/day) diet,
    which part of the nephron would be expected to secrete the most potassium?
    A) Proximal tubule
    B) Descending loop of Henle
    C) Ascending loop of Henle
    D) Early distal tubule
    E) Collecting tubules
A

E) Collecting tubules

68
Q
  1. Which of the following would you expect to find in a
    patient who has chronic diabetic ketoacidosis?
    A) Decreased renal HCO3
    − excretion, increased NH4+ excretion, increased plasma anion gap
    B) Increased respiration rate, decreased arterial
    Pco2, decreased plasma anion gap
    C) Increased NH4
    + excretion, increased plasma anion
    gap, increased urine pH
    D) Increased renal HCO3
    − production, increased
    NH4
    + excretion, decreased plasma anion gap
    E) Decreased urine pH, decreased renal HCO3
    − excretion, increased arterial Pco
A

A) Decreased renal HCO3

− excretion, increased NH4 + excretion, increased plasma anion gap

69
Q
  1. A patient has a creatinine clearance of 100 ml/min, a
    plasma K+ concentration of 4.0 mmol/L, a urine flow
    rate of 2.0 ml/min, and a urine K+ concentration of
    60 mmol/L. What is his approximate rate of potassium excretion?
    A) 0.12 mmol/min
    B) 0.16 mmol/min
    C) 0.32 mmol/min
    D) 8.0 mmol/min
    E) 120 mmol/min
    F) 400 mmol/min
A

A) 0.12 mmol/min

70
Q
  1. Using the indicator dilution method to assess body
    fluid volumes in a 40-year-old man weighing 70 kg,
    the inulin space is calculated to be 16 liters and 125
    I-albumin space is 4 liters. If 60% of his total body
    weight is water, what is his approximate interstitial
    fluid volume?
    A) 4 liters
    B) 12 liters
    C) 16 liters
    D) 26 liters
    E) 38 liters
    F) 42 liter
A

B) 12 liters

71
Q
118. What would tend to decrease plasma potassium concentration by causing a shift of potassium from the 
extracellular fluid into the cells?
A) Strenuous exercise
B) Aldosterone deficiency
C) Acidosis
D) β-adrenergic blockade
E) Insulin excess
A

E) Insulin excess

72
Q
  1. A 26-year-old construction worker is brought to the
    emergency department with a change in mental status
    after working a 10-hour shift on a hot summer day
    (average outside temperature was 97°F [36°C]). The
    man had been sweating profusely during the day but
    did not drink fluids. He has a fever of 102°F [39°C],
    a heart rate of 140 beats/min, and a blood pressure
    of 100/55 mm Hg in the supine position. Upon examination, he has no perspiration, appears to have dry
    mucous membranes, and is poorly oriented to person,
    place, and time. Assuming that his kidneys were normal yesterday, which set of hormone levels describes
    his condition, compared with normal?
    A) High ADH, high renin, low angiotensin II, low
    aldosterone
    B) Low ADH, low renin, low angiotensin II, low
    aldosterone
    C) High ADH, low renin, high angiotensin II, low
    aldosterone
    D) High ADH, high renin, high angiotensin II, high
    aldosterone
    E) Low ADH, high renin, low angiotensin II, high
    aldosterone
A

D) High ADH, high renin, high angiotensin II, high

aldosterone

73
Q
  1. Which change would tend to increase Ca2+ reabsorption in the renal tubule?
    A) Extracellular fluid volume expansion
    B) Increased plasma parathyroid hormone concentration
    C) Increased blood pressure
    D) Decreased plasma phosphate concentration
    E) Metabolic acidosis
A

B) Increased plasma parathyroid hormone concentration

74
Q
  1. A patient has the following laboratory values: arterial pH = 7.04, plasma HCO3
    − = 13 mEq/L, plasma
    chloride concentration = 120 mEq/L, arterial Pco2 =
    30 mm Hg, and plasma sodium = 141 mEq/L. What is
    the most likely cause of his acidosis?
    A) Emphysema
    B) Methanol poisoning
    C) Salicylic acid poisoning
    D) Diarrhea
    E) Diabetes mellitus
A

D) Diarrhea

75
Q
  1. A young man is found comatose, having taken an
    unknown number of sleeping pills an unknown time
    before. An arterial blood sample yields the following
    values: pH = 7.02, HCO3
    − = 14 mEq/L, and Pco2 =
    68 mm Hg. Which of the following describes this patient’s acid-base status most accurately?
    A) Uncompensated metabolic acidosis
    B) Uncompensated respiratory acidosis
    C) Simultaneous respiratory and metabolic acidosis
    D) Respiratory acidosis with partial renal compensation
    E) Respiratory acidosis with complete renal compensation
A

C) Simultaneous respiratory and metabolic acidosis

76
Q
  1. If the GFR suddenly decreases from 150 ml/min to
    75 ml/min and tubular fluid reabsorption simultaneously decreases from 149 ml/min to 75 ml/min, which
    change will occur (assuming that the changes in GFR
    and tubular fluid reabsorption are maintained)?
    A) Urine flow rate will decrease to 0
    B) Urine flow rate will decrease by 50%
    C) Urine flow rate will not change
    D) Urine flow rate will increase by 50%
A

A) Urine flow rate will decrease to 0

77
Q
  1. In a person with chronic respiratory acidosis who has
    partial renal compensation, you would expect to find
    which changes, compared with normal: ______ urinary
    excretion of NH4
    +; ______ plasma HCO3
    − concentration; and _____ urine pH.
    A) Increased, increased, decreased
    B) Increased, decreased, decreased
    C) No change in, increased, decreased
    D) No change in, no change in, decreased
    E) Increased, no change in, increased
A

A) Increased, increased, decreased

78
Q
  1. At which renal tubular sites would the concentration
    of creatinine be expected to be highest in a normally
    hydrated person?
    A) The concentration would be the same in all renal
    tubular segments because creatinine is neither
    secreted nor reabsorbed
    B) Glomerular filtrate
    C) End of the proximal tubule
    D) End of the loop of Henle
    E) Distal tubule
    F) Collecting duct
A

F) Collecting duct

79
Q
  1. Increases in both renal blood flow and GFR are caused
    by which mechanism?
    A) Dilation of the afferent arterioles
    B) Increased glomerular capillary filtration coefficient
    C) Increased plasma colloid osmotic pressure
    D) Dilation of the efferent arterioles
A

A) Dilation of the afferent arterioles

80
Q
130. If the cortical collecting tubule tubular fluid inulin 
concentration is 40 mg/100 ml and plasma concentration of inulin is 2.0 mg/100 ml, what is the approximate percentage of the filtered water that remains in 
the tubule at that point?
A) 0%
B) 2%
C) 5%
D) 10%
E) 20%
F) 100%
A

C) 5%

81
Q
  1. A 55-year-old male patient with hypertension has
    had his blood pressure reasonably well controlled by
    administration of a thiazide diuretic. At his last visit
    (6 months ago), his blood pressure was 130/75 mm Hg
    and his serum creatinine was 1 mg/100 ml. He has
    been exercising regularly for the past 2 years but recently has reported knee pain and began taking large
    amounts of a nonsteroidal anti-inflammatory drug.
    When he arrives at your office, his blood pressure is
    155/85 mm Hg and his serum creatinine is 2.5 mg/100
    ml. What best explains his increased serum creatinine
    level?
    A) Increased efferent arteriolar resistance that
    reduced GFR
    B) Increased afferent arteriolar resistance that
    reduced GFR
    C) Increased glomerular capillary filtration coefficient
    that reduced GFR
    D) Increased angiotensin II formation that decreased
    GFR
    E) Increased muscle mass due to the exercise
A

B) Increased afferent arteriolar resistance that

reduced GFR

82
Q
  1. An elderly patient reports muscle weakness and lethargy. A urine specimen reveals a Na+ concentration
    of 600 mmol/L and an osmolarity of 1200 mOsm/L.
    Additional laboratory tests provide the following information: plasma Na+ concentration = 167 mmol/L,
    plasma renin activity = 4 ng angiotensin I/ml/h (normal = 1), plasma ADH = 60 pg/ml (normal = 3 pg/ml),
    and plasma aldosterone = 15 ng/dl (normal = 6 ng/dl).
    What is the most likely reason for this patient’s hypernatremia?
    A) Dehydration caused by decreased fluid intake
    B) Syndrome of inappropriate ADH
    C) Nephrogenic diabetes insipidus
    D) Primary aldosteronism
    E) Renin-secreting tumor
A

A) Dehydration caused by decreased fluid intake